1
$\begingroup$

Suppose $Y$ is a Banach space and $X$ is a finite-dimensional subspace of $Y$. Further assume $T:X \rightarrow X$ is a linear operator which is power bounded from above and below, in other words there is $0 < c \leq 1 \leq C < \infty$ such that $c \cdot \|x\| \leq \|T^n(x)\| \leq C \cdot\|x\|$ for all $n \in \mathbb{N}$ and all $x \in X$.

Can $T$ be extended to a linear operator $\tilde{T}:Y \rightarrow Y$ which satisfies the same power bounds?

If it is not true in general, are there nice conditions which make it true? (It is important to my application that there is a lower bound.)


I apologize in advance if this is too trivial for Mathoverflow. I tried to look it up but couldn't find the answer. The impression I got is that extending linear operators is not as well known as extending linear functionals (Hahn-Banach). Also, while this is for an analysis paper, I am not an analyst. Hence I am not always sure what is common knowledge and what isn't. Thanks!

$\endgroup$

1 Answer 1

3
$\begingroup$

A finite-dimensional subspace is complemented, so we can write $Y = X \oplus Z$ for some closed subspace $Z$. There are constants $m$ and $M$ such that for all $x \in X$ and $z \in Z$, $m (\|x\| + \|z\|) \le \|x + z\| \le M (\|x\| + \|z\|)$. Extend $T$ to $\overline{T}$ so that $\overline{T} = I$ on $Z$. Then $\overline{T}$ is power-bounded above and below (though not necessarily with the same constants as $T$): $$ \|\overline{T}^n (x + z)\| = \|T^n x + z\| \le M (\|T^n x\| + \|z\|) \le C M (\|x\| + \|z\|) \le \frac{c M}{m} \|x + z\| $$ and similarly in the other direction.

It can't always be done with the same constants. Consider $Y = {\mathbb R}^3$ with the norm $\|(x,y,z)\| = \max(\sqrt{x^2 + y^2}, |x+z|)$, and $X = \{(x,y,0): x,y \in {\mathbb R}\}$. Note that $\|(x,y,0)\| = \sqrt{x^2 + y^2}$. Let $T: X \to X$ be a rotation $(x,y,0) \to (\cos(\theta) x + \sin(\theta) y, -\sin(\theta) x + \cos(\theta) y, 0)$ where $\theta$ is not an integer multiple of $\pi$. This is an isometry (so $c = C = 1$), but has no extension to an isometry of $Y$ because $(\pm 1, 0, 0)$ are the only points of $X$ that are extreme points of the unit ball of $Y$.

$\endgroup$
5
  • $\begingroup$ Thank you! What determines the bounds $m$ and $M$? I don't really need the same bounds for $T$ and $\tilde{T}$, but I do need some uniformity in deriving the new bounds from the previous ones (and the subspace $X$ can change). $\endgroup$
    – Jason Rute
    May 2, 2013 at 6:51
  • $\begingroup$ I did some more thinking. While $M$ can be 1, it seems that without more information $m$ can be arbitrary small depending on the choice of $Y$ and $X$. Oh well... $\endgroup$
    – Jason Rute
    May 2, 2013 at 8:15
  • 2
    $\begingroup$ I don't know, how good estimate you need, but one can always take $m$ to be equal to $C\sqrt{dimX}^{-1}$ (Kadec-Snobar theorem, $C$ is universal). $\endgroup$ May 2, 2013 at 8:56
  • $\begingroup$ Mateusz, my $X$'s grow in dimension so I don't think this will help. Also, if it matters, the $X$'s are all isomorphic (informally so) to $\ell^n_p$ where $n$ grows in size. But thank you for the reference! $\endgroup$
    – Jason Rute
    May 2, 2013 at 10:16
  • 4
    $\begingroup$ In general you cannot do better than the projection constant of $\ell_p^n$, which is of order $n^{1/p}$ when $p \ge 2$ and $n^{1/2}$ otherwise. To see that, look at $Y= \ell_p^n \oplus_p C(K)$ and take an isometric copy $E$ of $\ell_p^n$ in $C(K)$. On $\ell_p^n \oplus_p E$ let $T$ be an isometry that maps $\{0\} \oplus E$ to $\ell_p^n \oplus \{0\}$ and $\ell_p^n \oplus \{0\}$ to $\{0\} \oplus E$. $\endgroup$ May 2, 2013 at 13:06

Your Answer

By clicking “Post Your Answer”, you agree to our terms of service and acknowledge you have read our privacy policy.

Not the answer you're looking for? Browse other questions tagged or ask your own question.